Difference between revisions of "2000 AIME I Problems/Problem 2"
m |
(→Solution 2) |
||
(11 intermediate revisions by 7 users not shown) | |||
Line 1: | Line 1: | ||
== Problem == | == Problem == | ||
− | Let <math>u</math> and <math>v</math> be | + | Let <math>u</math> and <math>v</math> be [[integer]]s satisfying <math>0 < v < u</math>. Let <math>A = (u,v)</math>, let <math>B</math> be the [[reflection]] of <math>A</math> across the line <math>y = x</math>, let <math>C</math> be the reflection of <math>B</math> across the y-axis, let <math>D</math> be the reflection of <math>C</math> across the x-axis, and let <math>E</math> be the reflection of <math>D</math> across the y-axis. The area of [[pentagon]] <math>ABCDE</math> is <math>451</math>. Find <math>u + v</math>. |
− | == Solution == | + | == Solutions == |
− | {{solution}} | + | === Solution 1 === |
+ | <center><asy> | ||
+ | pointpen = black; pathpen = linewidth(0.7) + black; size(180); | ||
+ | pair A=(11,10), B=(10,11), C=(-10, 11), D=(-10, -11), E=(10, -11); | ||
+ | D(D(MP("A\ (u,v)",A,(1,0)))--D(MP("B",B,N))--D(MP("C",C,N))--D(MP("D",D))--D(MP("E",E))--cycle); | ||
+ | D((-15,0)--(15,0),linewidth(0.6),Arrows(5)); D((0,-15)--(0,15),linewidth(0.6),Arrows(5)); D((-15,-15)--(15,15),linewidth(0.6),Arrows(5)); | ||
+ | </asy></center> <!-- Asymptote replacement for Image:2000_I_AIME-2.png --> | ||
+ | |||
+ | Since <math>A = (u,v)</math>, we can find the coordinates of the other points: <math>B = (v,u)</math>, <math>C = (-v,u)</math>, <math>D = (-v,-u)</math>, <math>E = (v,-u)</math>. If we graph those points, we notice that since the latter four points are all reflected across the x/y-axis, they form a rectangle, and <math>ABE</math> is a triangle. The area of <math>BCDE</math> is <math>(2u)(2v) = 4uv</math> and the area of <math>ABE</math> is <math>\frac{1}{2}(2u)(u-v) = u^2 - uv</math>. Adding these together, we get <math>u^2 + 3uv = u(u+3v) = 451 = 11 \cdot 41</math>. Since <math>u,v</math> are positive, <math>u+3v>u</math>, and by matching factors we get either <math>(u,v) = (1,150)</math> or <math>(11,10)</math>. Since <math>v < u</math> the latter case is the answer, and <math>u+v = \boxed{021}</math>. | ||
+ | |||
+ | === Solution 2 === | ||
+ | We find the coordinates like in the solution above: <math>A = (u,v)</math>, <math>B = (v,u)</math>, <math>C = (-v,u)</math>, <math>D = (-v,-u)</math>, <math>E = (v,-u)</math>. Then we notice pentagon <math>ABCDE</math> fits into a rectangle of side lengths <math>(u+v)</math> and <math>(2u)</math>, giving us two triangles, each with hypotenuse <math>AB</math> and <math>BE</math>. First, we can solve for the first triangle. Using the coordinates of <math>A</math> and <math>B</math>, we discover the side lengths are both <math>(u-v)</math>, so the area of the triangle of hypotenuse <math>AB</math> is <math>\frac{1}{2}(u-v)^2</math>. Next, we can solve for the second triangle. Using the coordinates of <math>A</math> and <math>E</math>, we discover the side lengths are <math>(u-v)</math> and <math>(u+v)</math>, so the area of the triangle of hypotenuse <math>AE</math> is <math>\frac{1}{2}(u-v)(u+v) = \frac{1}{2}(u^2-v^2)</math>. Now, let’s subtract the area of these 2 triangles from the rectangle giving us <math>(u+v)(2u)-\frac{1}{2}(u-v)^2-\frac{1}{2}(u^2-v^2)=451 —> u^2+3uv=451 -> u(u+3v)=451</math>. Next, we take note of the fact that <math>u</math> and <math>u+3v</math> are both factors of 451, and since both <math>u</math> and <math>v</math> are positive integers, <math>u+3v</math> must be greater than <math>u</math>, thus giving us two cases, where either <math>u=1</math> or <math>u=11</math>. After trying both, the only working pair of <math>(u,v)</math> where both <math>u</math> and <math>v</math> are integers are <math>u=11</math> and <math>v=10</math>, thus meaning <math>u + v =</math> <math>\boxed{021}</math> | ||
+ | |||
+ | ~Aeioujyot | ||
+ | |||
+ | === Solution 3 === | ||
+ | We find the coordinates like in the solution above: <math>A = (u,v)</math>, <math>B = (v,u)</math>, <math>C = (-v,u)</math>, <math>D = (-v,-u)</math>, <math>E = (v,-u)</math>. Then we apply the [[Shoelace Theorem]]. <cmath>A = \frac{1}{2}[(u^2 + vu + vu + vu + v^2) - (v^2 - uv - uv - uv -u^2)] = 451</cmath> <cmath>\frac{1}{2}(2u^2 + 6uv) = 451</cmath> <cmath>u(u + 3v) = 451</cmath> | ||
+ | |||
+ | This means that <math>(u,v) = (11, 10)</math> or <math>(1,150)</math>, but since <math>v < u</math>, then the answer is <math>\boxed{021}</math> | ||
== See also == | == See also == | ||
{{AIME box|year=2000|n=I|num-b=1|num-a=3}} | {{AIME box|year=2000|n=I|num-b=1|num-a=3}} | ||
+ | |||
+ | [[Category:Intermediate Geometry Problems]] | ||
+ | [[Category:Intermediate Number Theory Problems]] | ||
+ | {{MAA Notice}} |
Latest revision as of 13:31, 22 October 2024
Problem
Let and be integers satisfying . Let , let be the reflection of across the line , let be the reflection of across the y-axis, let be the reflection of across the x-axis, and let be the reflection of across the y-axis. The area of pentagon is . Find .
Solutions
Solution 1
Since , we can find the coordinates of the other points: , , , . If we graph those points, we notice that since the latter four points are all reflected across the x/y-axis, they form a rectangle, and is a triangle. The area of is and the area of is . Adding these together, we get . Since are positive, , and by matching factors we get either or . Since the latter case is the answer, and .
Solution 2
We find the coordinates like in the solution above: , , , , . Then we notice pentagon fits into a rectangle of side lengths and , giving us two triangles, each with hypotenuse and . First, we can solve for the first triangle. Using the coordinates of and , we discover the side lengths are both , so the area of the triangle of hypotenuse is . Next, we can solve for the second triangle. Using the coordinates of and , we discover the side lengths are and , so the area of the triangle of hypotenuse is . Now, let’s subtract the area of these 2 triangles from the rectangle giving us . Next, we take note of the fact that and are both factors of 451, and since both and are positive integers, must be greater than , thus giving us two cases, where either or . After trying both, the only working pair of where both and are integers are and , thus meaning
~Aeioujyot
Solution 3
We find the coordinates like in the solution above: , , , , . Then we apply the Shoelace Theorem.
This means that or , but since , then the answer is
See also
2000 AIME I (Problems • Answer Key • Resources) | ||
Preceded by Problem 1 |
Followed by Problem 3 | |
1 • 2 • 3 • 4 • 5 • 6 • 7 • 8 • 9 • 10 • 11 • 12 • 13 • 14 • 15 | ||
All AIME Problems and Solutions |
The problems on this page are copyrighted by the Mathematical Association of America's American Mathematics Competitions.